Hvor mange kvadrater finnes i følgen?

Her kan brukere av forum utfordre hverandre med morsomme oppgaver og nøtter man ønsker å dele med andre. Dette er altså ikke et sted for desperate skrik om hjelp, de kan man poste i de andre forumene, men et sted for problemløsing på tvers av trinn og fag.

Moderatorer: Vektormannen, espen180, Aleks855, Solar Plexsus, Gustav, Nebuchadnezzar, Janhaa

Svar
Brahmagupta
Guru
Guru
Innlegg: 628
Registrert: 06/08-2011 01:56

La [tex]\{a_n\}_{n\geq1}[/tex] være en følge med [tex]a_1=1[/tex] og
[tex]a_{n+1}=\lfloor a_n+\sqrt{a_n}+\frac12 \rfloor[/tex]

Der [tex]\lfloor x \rfloor[/tex] står for det største heltallet mindre eller lik [tex]x[/tex].
Finn alle [tex]n\leq 2013[/tex] slik at [tex]a_n[/tex] er et kvadrattall.
Aleks855
Rasch
Rasch
Innlegg: 6862
Registrert: 19/03-2011 15:19
Sted: Trondheim
Kontakt:

EDIT: Fant svaret, men kan ikke bevise det.
Bilde
Brahmagupta
Guru
Guru
Innlegg: 628
Registrert: 06/08-2011 01:56

Ja, men kan du bevise det? ;)
Aleks855
Rasch
Rasch
Innlegg: 6862
Registrert: 19/03-2011 15:19
Sted: Trondheim
Kontakt:

Slike oppgaver er egentlig litt over mitt hode... foreløpig. Hehe :) Jeg fjerna svaret, bare for å unngå å spoile.
Bilde
jhoe06
Cantor
Cantor
Innlegg: 107
Registrert: 07/12-2011 14:44

Hvis vi skriver opp de første leddene i rekken, ser vi et tydelig mønster: differansen mellom leddene lager rekken 1, 1, 2, 2, 3, 3, ... . Hvis dette mønsteret viser seg å holde for alle n, kan vi bruke identiteten [tex]\sum_{n=1}^k n =\frac{1}{2} k(k+1)[/tex] til å finne en formel for [tex]a_n[/tex]. Vi kan skrive

[tex]a_n = 1 + k + k^2[/tex] dersom n er et oddetall, [tex]k = \frac{n-1}{2}[/tex]
[tex]a_n = 1 + k^2[/tex] dersom n er et partall, [tex]k = \frac{n}{2}[/tex]

Den nøyaktige verdien til k er ikke så viktig, siden k er et heltall. Forutsatt at mønsteret stemmer, er det enkelt å vise at n = 1 gir det eneste kvadratet i følgen, siden for [tex]k > 0[/tex] har vi at [tex]k^2 < 1 + k + k^2 < (1+k)^2[/tex], og at [tex]1+k^2[/tex] ikke er et kvadrat.

Det gjenstår nå å vise at mønsteret vi har observert faktisk stemmer. Den første observasjonen vi gjør er at [tex]a_{n+1} = a_n + \lfloor \sqrt{a_n} + \frac{1}{2} \rfloor[/tex], siden [tex]a_n[/tex] er et heltall. For å gjøre skriving og lesing noe enklere, setter vi [tex]b_n = \lfloor \sqrt{a_n} + \frac{1}{2} \rfloor[/tex] slik at [tex]a_{n+1} = a_n + b_n[/tex]. Da har vi også at [tex]b_{n+1}=\lfloor \sqrt{a_n + b_n} + \frac{1}{2} \rfloor[/tex].

Det vi ønsker å vise er (1) at dersom [tex]\sqrt{a_n} + \frac{1}{2}[/tex] er i nedre del av [tex][b_n, b_n + 1)[/tex], så har vi at [tex]b_{n+1}=b_n[/tex] og at [tex]\sqrt{a_n + b_n} + \frac{1}{2}[/tex] er i øvre del av [tex][b_n, b_n+1)[/tex], og (2) at dersom [tex]\sqrt{a_n} + \frac{1}{2}[/tex] er i øvre del av [tex][b_n, b_n + 1)[/tex] så har vi at [tex]b_{n+1}=b_n+1[/tex] og at [tex]\sqrt{a_n + b_n} + \frac{1}{2}[/tex] er i nedre del av [tex][b_{n+1}, b_{n+1} + 1)[/tex].

For (1) skal vi vise

[tex]b_n \leq \sqrt{a_n} + \frac{1}{2} < b_n + \frac{1}{2} + \epsilon_n \implies b_n + \frac{1}{2} + \epsilon_n \leq \sqrt{a_n + b_n} + \frac{1}{2} < b_n + 1[/tex]

For den venstre ulikheten har vi

[tex]b_n^2 - b_n + \frac{1}{4} \leq a_n \implies b_n^2 + \frac{1}{4} \leq a_n + b_n \implies b_n + \frac{1}{2} + \epsilon_n \leq \sqrt{a_n + b_n}[/tex]

hvor [tex]\epsilon_n = \sqrt{b_n^2 + \frac{1}{4}}-b_n[/tex]

For den høyre ulikheten har vi

[tex]a_n < (b_n+\epsilon_n)^2 = b_n^2 + \frac{1}{4} \implies a_n + b_n < (b_n + \frac{1}{2})^2 \implies \sqrt{a_n + b_n} + \frac{1}{2} < b_n + 1[/tex]

For (2) skal vi vise

[tex]b_n + \frac{1}{2} + \epsilon_n \leq \sqrt{a_n} + \frac{1}{2} < b_n + 1 \implies b_n + 1 \leq \sqrt{a_n + b_n} + \frac{1}{2} < b_n + \frac{3}{2}[/tex]

For den venstre ulikheten har vi

[tex](b_n + \epsilon_n)^2 = b_n^2 + \frac{1}{4} \leq a_n \implies (b_n+\frac{1}{2})^2 \leq a_n + b_n \implies b_n + 1 \leq \sqrt{a_n+b_n} + \frac{1}{2}[/tex]

For den høyre ulikheten har vi

[tex]a_n < (b_n + \frac{1}{2})^2 = b_n^2 + b_n + \frac{1}{4} \implies a_n + b_n < b_n^2 + 2b_n + 1 \implies \sqrt{a_n + b_n} + \frac{1}{2} < b+1+\frac{1}{2}[/tex]

Ved induksjon vil dermed mønsteret vi har observert, nemlig at [tex]b_1, b_2, b_3, b_4 ... = 1, 1, 2, 2, ...[/tex] fortsette for evig og alltid, og vi har dermed at [tex]a_1[/tex] er det eneste kvadratet i følgen.
Svar